Kurs:Grundkurs Mathematik/Teil II/3/Klausur/latex

Aus Wikiversity

%Daten zur Institution

%\input{Dozentdaten}

%\renewcommand{\fachbereich}{Fachbereich}

%\renewcommand{\dozent}{Prof. Dr. . }

%Klausurdaten

\renewcommand{\klausurgebiet}{ }

\renewcommand{\klausurtyp}{ }

\renewcommand{\klausurdatum}{ . 20}

\klausurvorspann {\fachbereich} {\klausurdatum} {\dozent} {\klausurgebiet} {\klausurtyp}


%Daten für folgende Punktetabelle


\renewcommand{\aeins}{ 3 }

\renewcommand{\azwei}{ 3 }

\renewcommand{\adrei}{ 4 }

\renewcommand{\avier}{ 3 }

\renewcommand{\afuenf}{ 4 }

\renewcommand{\asechs}{ 4 }

\renewcommand{\asieben}{ 3 }

\renewcommand{\aacht}{ 1 }

\renewcommand{\aneun}{ 4 }

\renewcommand{\azehn}{ 3 }

\renewcommand{\aelf}{ 2 }

\renewcommand{\azwoelf}{ 2 }

\renewcommand{\adreizehn}{ 7 }

\renewcommand{\avierzehn}{ 4 }

\renewcommand{\afuenfzehn}{ 5 }

\renewcommand{\asechzehn}{ 6 }

\renewcommand{\asiebzehn}{ 4 }

\renewcommand{\aachtzehn}{ 3 }

\renewcommand{\aneunzehn}{ 65 }

\renewcommand{\azwanzig}{ }

\renewcommand{\aeinundzwanzig}{ }

\renewcommand{\azweiundzwanzig}{ }

\renewcommand{\adreiundzwanzig}{ }

\renewcommand{\avierundzwanzig}{ }

\renewcommand{\afuenfundzwanzig}{ }

\renewcommand{\asechsundzwanzig}{ }

\punktetabelleachtzehn


\klausurnote

\newpage


\setcounter{section}{0}




\inputaufgabegibtloesung
{3}
{

Definiere die folgenden \zusatzklammer {kursiv gedruckten} {} {} Begriffe. \aufzaehlungsechs{Die \stichwort {Matrizenmultiplikation} {.}

}{Ein \stichwort {Repräsentantensystem} {} zu einer Äquivalenzrelation $\sim$ auf einer Menge $M$.

}{Eine \stichwort {Cauchy-Folge} {} ${ \left( x_n \right) }_{n \in \N }$ in einem angeordneten Körper $K$.

}{Die \stichwort {Eulersche Zahl} {.}

}{Der \stichwort {Sinus} {} zu einem Winkel $\alpha$.

}{Ein \stichwort {Laplace-Raum} {.} }

}
{} {}




\inputaufgabegibtloesung
{3}
{

Formuliere die folgenden Sätze. \aufzaehlungdrei{Der Festlegungssatz für lineare Abbildungen \maabbdisp {\varphi} {K^n} {K^m } {.}}{Der Satz über die Irrationalität von Wurzeln aus natürlichen Zahlen.}{Der \stichwort {Satz von Vieta} {.}}

}
{} {}




\inputaufgabegibtloesung
{4}
{

Bestimme die \definitionsverweis {inverse Matrix}{}{} zu
\mathdisp {\begin{pmatrix} 2 & 4 & 0 \\ -1 & 0 & 3 \\0 & 1 & 1 \end{pmatrix}} { . }

}
{} {}




\inputaufgabegibtloesung
{3}
{

Zeige, dass die folgende Relation eine \definitionsverweis {Äquivalenzrelation}{}{} auf $\Z$ ist:
\mathdisp {x \sim y, \text{ falls } 7 \text{ ein Teiler von } x-y \text{ ist}} { . }

}
{} {}




\inputaufgabegibtloesung
{4 (1+3)}
{

\aufzaehlungzwei {Gibt es eine Primzahl $x$ derart, dass auch
\mathl{x+10}{} und
\mathl{x+20}{} Primzahlen sind? } {Gibt es mehr als eine Primzahl $x$ derart, dass auch
\mathl{x+10}{} und
\mathl{x+20}{} Primzahlen sind? }

}
{} {}




\inputaufgabegibtloesung
{4}
{

Beweise das Kernkriterium für die Injektivität eines Gruppenhomomorphismus \maabbdisp {\varphi} {G} {H } {.}

}
{} {}




\inputaufgabegibtloesung
{3}
{

Es sei
\mathl{c \in K_+}{} ein Element in einem angeordneten Körper $K$ und sei
\mathl{{ \left( x_n \right) }_{n \in \N }}{} die \definitionsverweis {Heron-Folge}{}{} zur Berechnung von $\sqrt{c}$ mit dem Startwert
\mathl{x_0 \in K_+}{.} Es sei
\mathl{u \in K_+}{,}
\mavergleichskette
{\vergleichskette
{d }
{ = }{c \cdot u^2 }
{ }{ }
{ }{ }
{ }{ }
} {}{}{,}
\mavergleichskette
{\vergleichskette
{y_0 }
{ = }{ u x_0 }
{ }{ }
{ }{ }
{ }{ }
} {}{}{} und
\mathl{{ \left( y_n \right) }_{n \in \N }}{} die Heron-Folge zur Berechnung von $\sqrt{d}$ mit dem Startwert $y_0$. Zeige
\mavergleichskettedisp
{\vergleichskette
{y_n }
{ =} {u x_n }
{ } { }
{ } { }
{ } { }
} {}{}{} für alle
\mathl{n\in \N}{.}

}
{} {}




\inputaufgabegibtloesung
{1}
{

Ist die reelle Zahl
\mathdisp {\sqrt{ { \frac{ 1 }{ 11 } } }} { }
rational?

}
{} {}




\inputaufgabegibtloesung
{4}
{

Zeige, dass eine \definitionsverweis {konvergente Folge}{}{}
\mathl{{ \left( x_n \right) }_{n \in \N }}{} in einem \definitionsverweis {angeordneten Körper}{}{} $K$ \definitionsverweis {beschränkt}{}{} ist.

}
{} {}




\inputaufgabegibtloesung
{3}
{

Man gebe ein Beispiel für eine konvergente Folge
\mathl{{ \left( x_n \right) }_{n \in \N }}{} in einem \definitionsverweis {angeordneten Körper}{}{} $K$, die in einem größeren angeordneten Körper
\mavergleichskettedisp
{\vergleichskette
{K }
{ \subseteq} {L }
{ } { }
{ } { }
{ } { }
} {}{}{} nicht konvergiert.

}
{} {}




\inputaufgabegibtloesung
{2}
{

Schreibe die Menge
\mathdisp {]-3,-2[ \, \cup \, \{7\} \, \cup \, { \left( [- { \frac{ 5 }{ 2 } } , - { \frac{ 1 }{ 3 } } ] \, \setminus \, ] - { \frac{ 4 }{ 3 } }, -1 ] \right) } \, \cup \, [1, { \frac{ 7 }{ 3 } } ] \, \cup \, [- { \frac{ 1 }{ 2 } } , { \frac{ 6 }{ 5 } } [ \, \cup \, { \left( \, ]-7,-6] \cap \R_+ \right) }} { }
als eine Vereinigung von möglichst wenigen disjunkten Intervallen.

}
{} {}




\inputaufgabegibtloesung
{2}
{

Bestimme eine Symmetrieachse für den Graphen der Funktion \maabbeledisp {f} {\R} {\R } {x} {x^2-5x-9 } {.}

}
{} {}




\inputaufgabegibtloesung
{7}
{

Beweise den Satz über die Division mit Rest im Polynomring
\mathl{K[X]}{} über einem Körper $K$.

}
{} {}




\inputaufgabegibtloesung
{4}
{

Es sei
\mathl{z \in \R}{} derart, dass es ein Polynom $P \neq 0$ mit rationalen Koeffizienten und mit
\mavergleichskettedisp
{\vergleichskette
{P(z) }
{ =} {0 }
{ } { }
{ } { }
{ } { }
} {}{}{} gibt. Zeige, dass man
\mavergleichskettedisp
{\vergleichskette
{z }
{ =} { { \frac{ u }{ v } } }
{ } { }
{ } { }
{ } { }
} {}{}{} schreiben kann, wobei $v$ eine positive natürliche Zahl ist und es zu $u$ ein normiertes Polynom $Q$ mit ganzzahligen Koeffizienten und mit
\mavergleichskettedisp
{\vergleichskette
{Q(u) }
{ =} {0 }
{ } { }
{ } { }
{ } { }
} {}{}{} gibt.

}
{} {}




\inputaufgabegibtloesung
{5 (1+1+3)}
{

Wir betrachten das Polynom
\mavergleichskettedisp
{\vergleichskette
{P }
{ =} {1+X + { \frac{ 1 }{ 2 } } X^2 +{ \frac{ 1 }{ 6 } } X^3 }
{ } { }
{ } { }
{ } { }
} {}{}{.} \aufzaehlungdrei{Berechne die Werte von $P$ an den Stellen
\mathl{-2,-1,0,1,2}{.} }{Skizziere den Graphen von $P$ auf dem Intervall
\mathl{[-2,2]}{.} Gibt es einen Bezug zur Exponentialfunktion $e^x$? }{Bestimme eine Nullstelle von $P$ innerhalb von
\mathl{[-2,2]}{} mit einem Fehler von maximal
\mathl{{ \frac{ 1 }{ 4 } }}{.} }

}
{} {}




\inputaufgabegibtloesung
{6}
{

Berechne die Schnittpunkte der beiden Kreise \mathkor {} {K_1} {und} {K_2} {,} wobei $K_1$ den Mittelpunkt
\mathl{(3,4)}{} und den Radius $6$ und $K_2$ den Mittelpunkt $(-8,1)$ und den Radius $7$ besitzt.

}
{} {}




\inputaufgabegibtloesung
{4}
{

Es werden unabhängig voneinander zwei Zahlen
\mathl{x,y}{} aus
\mathl{\{1,2 , \ldots , 10\}}{} gezogen. Bestimme die Wahrscheinlichkeit, dass
\mavergleichskettedisp
{\vergleichskette
{xy }
{ \geq} {48 }
{ } { }
{ } { }
{ } { }
} {}{}{} ist.

}
{} {}




\inputaufgabegibtloesung
{3}
{

Es sei $M$ ein \definitionsverweis {endlicher Wahrscheinlichkeitsraum}{}{} und es seien
\mavergleichskette
{\vergleichskette
{A,B }
{ \subseteq }{M }
{ }{ }
{ }{ }
{ }{ }
} {}{}{} Ereignisse mit
\mavergleichskette
{\vergleichskette
{P(A \cap B) }
{ \neq }{0 }
{ }{ }
{ }{ }
{ }{ }
} {}{}{} und mit
\mavergleichskette
{\vergleichskette
{P(A),P(B) }
{ < }{1 }
{ }{ }
{ }{ }
{ }{ }
} {}{}{.} Zeige
\mavergleichskettedisp
{\vergleichskette
{ { \frac{ P( B {{|}} A) }{ P( M \setminus B {{|}} A) } } }
{ =} { { \frac{ P( B ) }{ P( M \setminus B ) } } \cdot { \frac{ P( A {{|}} B) }{ P( A {{|}} M \setminus B ) } } }
{ } { }
{ } { }
{ } { }
} {}{}{.}

}
{} {}